0 Daumen
2,8k Aufrufe

Kann mir hier jemand helfen? Mir fehlt der Ansatz.

Sei \( \mathbb{K} \) ein Körper und \( f: \mathrm{M}_{22}(\mathbb{K}) \rightarrow \mathbb{K} \) definiert durch \( \left(\begin{array}{cc}{a} & {b} \\ {c} & {d}\end{array}\right) \mapsto a d-b c \) für alle \( \left(\begin{array}{cc}{a} & {b} \\ {c} & {d}\end{array}\right) \in \)
\( \mathrm{M}_{22}(\mathrm{K}) \)

Beweisen oder widerlegen Sie folgende Behauptungen.

1. Die Abbildung \( f \) ist surjektiv.
2. Die Abbildung \( f \) ist nicht injektiv.
3. Für alle \( A, B \in \mathrm{M}_{22}(\mathbb{K}) \) gilt \( f(A B)=f(A) f(B) \)
4. Für alle \( A, B \in \mathrm{M}_{22}(\mathrm{K}) \) gilt \( f(A+B)=f(A)+f(B) \)

Avatar von

1 Antwort

0 Daumen

1. Ja. Sei a Element K. ==> a ist als Blid möglich.

Matrix M=

(a 0

0 1)

|--> a             , a Element K.

hat als Bild alle Elemente von K.

2. ja. Nicht injektiv.  EDIT: Beachte das allgemeinere Gegenbeispiel von Nick im Kommentar.

M1:=

(1 0

0 2)

und

M2:=

(2 0

4 1)

werden beide auf 2 abgebildet, aber M1≠M2.

Avatar von 162 k 🚀

4. ist falsch:

M3:=

( 1 3

0  2)

I----> 2

und

M4:=

(0 0

0 1)

|----> 0

M3 + M4 =

(1 3

0 3)

|----> 3

Aber 2 + 0 ≠ 3. q.e.d. 4. ist falsch.

3. stimmt.

A:=

( a b

c d)

|----> ad - bc

B:=

(u v

w x)

|--> ux - wv

(ad - bc)(ux - wv)  = adux - adwv - bcux + bcwv

AB =

( (au + bw)    (av + bx)

(cu + dw)     (cv + dx) )

|---->   (au + bw)(cv + dx) - (cu + dw)(av + bx)

= acuv + bcvw + adux + bdwx - acuv - advw - bcux - bdwx 

=  bcvw + adux - advw - bcux 

q.e.d. 

Die Begründung zu 2. kann man so nicht machen. Man weiß ja gar nichts über den Körper, also auch nicht, ob er die 2 und die 4 enthält und wie die Multiplikation und Addition dieser Elemente definiert ist.

Ein Körper enthält immer ein Null- und ein Einselement, deswegen solte man auch nur die benutzen. Z.B. \(A=\begin{pmatrix}1&1\\1&1\end{pmatrix}, B=\begin{pmatrix}0&0\\0&0\end{pmatrix}\). Dann ist \(f(A)=f(B)=0\).

Nick: Besten Dank.

Du zeigst, dass f in keinem Körper injektiv ist. Das ist natürlich allgemeiner und daher sicher besser.

Ich habe mE gezeigt, dass f nicht in jedem Körper injektiv ist.

Ein anderes Problem?

Stell deine Frage

Willkommen bei der Mathelounge! Stell deine Frage einfach und kostenlos

x
Made by a lovely community